LSAT and Law School Admissions Forum

Get expert LSAT preparation and law school admissions advice from PowerScore Test Preparation.

User avatar
 Dave Killoran
PowerScore Staff
  • PowerScore Staff
  • Posts: 5853
  • Joined: Mar 25, 2011
|
#41187
Complete Question Explanation
(The complete setup for this game can be found here: lsat/viewtopic.php?t=15652)

The correct answer choice is (C)

This can be a difficult question. The question stem stipulates that M lives west of K, and so M must live in the third house. The most logical place to look for an answer then, would be with variables that are forcibly separated. P and L must both live east of K, and so a combination of M and P or M and L would be correct.

Specifically, when M lives in the third house, and we know already that P must live in the fifth or sixth house, we can infer that R cannot live next to both M and P because K will serve to separate the houses. Thus, answer choice (C) is correct.
 S_Hernandez52
  • Posts: 13
  • Joined: Feb 28, 2017
|
#33489
I could not get number 10 is it possible to see the master set up and the mini local set up to see how to better attack this question. I somehow had to draw multiple scenarios that got jumbled up and was hard to see which one's R can't be next to. Furthermore, when it says CANNOT be next to does this question mean something that it can't be RKL, KRL, LKR. Hopefully this makes sense. Once again thank you for your help!
 Charlie Melman
PowerScore Staff
  • PowerScore Staff
  • Posts: 85
  • Joined: Feb 10, 2017
|
#33508
Hi S,

It's tough to draw a diagram here on the forum, so please see my hand-drawn diagram by clicking here. I split the main diagram into two diagrams based on rule 2, because there are only two places where M could go. Rule 2 says that M and K will be next to each other, but can swap their order. Please let me know if you need anything clarified.

There is no one sub-diagram for Question 10. The thing to note here is that if R is next to two things at once, he must be in between them. So each answer choice can be visually represented as [ ____ R ____ ], where each blank is filled by the two items given in the answer choice.

Hope this helps!
 S_Hernandez52
  • Posts: 13
  • Joined: Feb 28, 2017
|
#33542
Thank you this did help!
 lety.gordill@gmail.com
  • Posts: 7
  • Joined: Apr 19, 2021
|
#91504
Hello,

I really struggled through this question, and I was wondering if it would be possible to be walked through each of the answer choices to understand why those are wrong.

Thank you
 Adam Tyson
PowerScore Staff
  • PowerScore Staff
  • Posts: 5153
  • Joined: Apr 14, 2011
|
#91516
I'll do my best, lety!

First, with M west of K, and also next to K per the original rules, that puts M is the 3rd spot. K is, as always, 4th. The starting setup for the question, then, is like this:

_ _ M K _ _ _

Add to that the fact that P is after K and before L, and that puts P-L both East of (after) K. So in those last three spaces you have P before L, and some other variable is also there, but we cannot be sure which variable it is or in which space.

The question now is about a pair of variables whom R cannot be next to both. In other words, which pair can R NOT be sandwiched directly between? We know from the rules that R cannot be 1st or last, but in this scenario that still leaves 2nd, 5th, or 6th. So let's run through the pairs that R COULD be between:

If R is 2nd, R will be directly between M (who is 3rd) and either N or O, depending on which of them is first. Thus, answers D and E are both wrong answers, because R CAN be next to those pairs.

If R is 5th, R will have to be directly between (next to both) K and P. So answer A is out because it CAN happen. We want a pair that R CANNOT be directly between.

If R is 6th, R will have to be between (next to both) P and L, and so answer B is out.

That just leaves answer C! There is no way for R to be next to BOTH M and P, because K is in the way. If R is next to M, then R cannot also be next to P, and if R is next to P, it cannot also be next to M. Thus, answer C is impossible, and must be the correct answer.

Finally, to help you see why all the wrong answers are wrong, here are possible solutions for each of them:

Answer A: NOMKRPL (K is next to both K and P)
Answer B: NOMKPRL (K is next to both L and P)
Answer D: ORMKPLN (R is next to both M and O
Answer E: NRMKOPL (R is next to both M and N

The trickiest part of this question may be interpreting "next to both" as meaning "directly between." You have to recognize that being next to both variables means being next to both at the same time. The only way to do that is to be right in between them!
 olenka.ballena@macaulay.cuny.edu
  • Posts: 17
  • Joined: Feb 16, 2022
|
#96811
Hi Powerscore,

I got this question right, but I think it may have taken me long, and I was wondering if there's a more time efficient to answer this question rather than going through all the possible scenarios where R could be placed (in compliance with the not laws, of course). If there's not a much more time efficient way, is there a way to distinguish which questions will require several diagrams versus ones that can be answered more directly?
 Adam Tyson
PowerScore Staff
  • PowerScore Staff
  • Posts: 5153
  • Joined: Apr 14, 2011
|
#97390
While my earlier explanation was based on testing the places where R could go, that was only to illustrate what made the wrong answers wrong. A more efficient way to approach this question would be to scan the answer choices and simply look for a pair of variables that must have at least one other variable between them. That is what prevents R from being next to both of them. Find that pair and you have the right answer!

Get the most out of your LSAT Prep Plus subscription.

Analyze and track your performance with our Testing and Analytics Package.